In the passage, the author is primarily concerned with

Julia on August 6, 2020

Why is E correct? Why is B incorrect?

.

Reply
Create a free account to read and take part in forum discussions.

Already have an account? log in

Shunhe on August 11, 2020

Hi @Julia!,

Thanks for the question! So we’re asked for the primary concern of the passage here. This is a good thing to try to come up with in our heads before looking at the answer choices—get a bird’s eye picture of the passage, and then compare that to our available options.

So what’s going on here? Well, we’re given an introductory passage about “hard” cases, and then we’re introduced to this debate. The next paragraph describes one side of that debate, which is Hart’s side. The following paragraph (third paragraph) then describes the other side of that passage, or Dworkin’s side of the argument, and how it differs from Hart’s side. Finally, the author reveals their opinion on the whole matter, telling us that “it would be a mistake, though, to dispute Hart’s theory of hard cases on this basis alone” (lines 46-47). So the whole passage kind of leads up to this last paragraph. And what are we told in the last paragraph? That Dworkin’s concept by itself isn’t enough to refute Hart. And that’s summed up pretty well by (E), that the passage as a whole has a purpose of showing that Dworkin’s concept of legal principles doesn’t form the basis for a successful attack on Hart’s theory of legally indeterminate cases. So since everything builds up to this point, it’s fair to say it’s the primary purpose, and (E) is the correct answer.

(B), on the other hand, is too strongly worded. Does the passage show that “hard” cases will ALWAYS exist in the practice of law, no matter what laws are written or how they’re applied? That goes beyond what the passage is arguing, it’s just way too strong. And in addition, that’s not the main discussion of the topic. It’s not like Hart is saying they do exist and Dworkin’s saying they don’t. Dworkin’s just talking about what happens when they do exist. So for those reasons, (B) is wrong.

Hope this helps! Feel free to ask any other questions that you might have.